Accepted at 7:51 p.m. Apr, 30, 2024 by herstein.jacob
Author: thomas.holmes
Co-authors: herstein.jacob
Related Note: 1571329545418 2
Rationale for change

UWorld says dose reduction is 1st line, if persistent then add low-potency anti-psychotic
Source: UWorld - Step 2 16164

Text Text
Extra
- First reduce the dose of the dopamine agonist, but consider adding a low-potency antipsychotic if psychotic symptoms continue (e.g., quetiapine, pimavanserin)

- High-potency antipsychotics (e.g., haloperidol) block D2 receptors more → ↑ extrapyramidal symptoms of Parkinson's disease
Lecture Notes
Empty field
Missed Questions
Empty field
Pathoma
Empty field
Boards and Beyond
Empty field
First Aid
Empty field
Sketchy
Empty field
Sketchy 2
Empty field
Sketchy Extra
Empty field
Picmonic
Empty field
Pixorize
Empty field
Physeo
Empty field
Bootcamp
Empty field
OME

Additional Resources
Empty field
One by one
Empty field
#AK_Step2_v12::#B&B::10_Neuro::03_Movement_Disorders::01_Parkinson's_Disease #AK_Step2_v12::#Resources_by_rotation::Psych::uworld::dorian #AK_Step2_v12::#UWorld::COMLEX::103278 #AK_Step2_v12::#UWorld::COMLEX::103189 #AK_Step2_v12::!Shelf::Psych::no_dupes::only_step2 #AK_Step2_v12::!Shelf::Psych::no_dupes #AK_Step2_v12::#Resources_by_rotation::Psych::nbme::form_6 #AK_Step2_v12::#AMBOSS::C8aqKm #AK_Step2_v12::!Shelf::#Cards_AnKing_Did::5psych #AK_Step2_v12::#UWorld::Step::16164 !AK_UpdateTags::Step2decks::Cheesy-Dorian-(M3)::Psychiatry::3-UWorld #AK_Step2_v12::#B&B::13_Psychiatry::04_Psychopharmacology::02_Antipsychotics::03_Atypical_antipsychotics #AK_Step2_v12::#Subjects::Neurology::11_Degenerative_Disorders::Parkinson_Disease #AK_Step2_v12::#NBME #AK_Step2_v12::#AMBOSS #AK_Step2_v12::#UWorld::Step::16051 #AK_Step2_v12::Original_decks::Dorian::psych::uw #AK_Step2_v12::#Subjects::Neurology::11_Degenerative_Disorders::Parkinson_Disease::Management #AK_Original_Decks::Step_2::Cheesy_Dorian_(M3) #AK_Step2_v12::#OME_banner #AK_Step2_v12::#UWorld::Step::12152 #AK_Step2_v12::#UWorld::COMLEX::102073